Medical Oncology Board Review Questions

html xmlns=”http://www.w3.org/1999/xhtml”>

21.


MEDICAL ONCOLOGY BOARD REVIEW QUESTIONS


Lawrence N. Shulman


QUESTION 1. A 63-year-old woman is 8 years status post-lumpectomy and radiation therapy for a 1.5cm, estrogen-receptor–positive, node-negative breast cancer for which she had received 5 years of tamoxifen stopped 3 years ago. She presents to your office with severe, localized back pain. Physical examination is normal including the neurological examination. The alkaline phosphatase is 330 U/L (elevated), and the CA27.29 is 156 (elevated). A bone scan is positive in several areas of the thoracic and lumbar spine as well as in several ribs. The course of action at this point should be:


    A. Combination chemotherapy


    B. Tamoxifen therapy


    C. Magnetic resonance imaging (MRI) scan of the spine


    D. Radiation therapy to areas of localized disease


    E. Stem cell supported transplantation


QUESTION 2. A 68-year-old man presents with back pain, anemia, and fevers. The patient has no lymphadenopathy or splenomegaly. Laboratory evaluation reveals Hct = 34%, platelet count = 89,000/mm3, and a total protein of 9.8 g/dL. The serum creatinine is 3.2 mg/dL, and the serum calcium is 12.3 mg/dL. Plain x-rays of the spine show generalized osteoporosis without focal defects. The following best explains the situation:


    A. Fever is a worrisome sign, and infection is a life-threatening risk for patients with this diagnosis.


    B. Renal failure is uncommon and not likely to worsen.


    C. Myeloma cannot be the diagnosis because lytic bone lesions are not seen.


    D. Waldenstrom’s macroglobulinemia is never associated with lymphadenopathy and/or splenomegaly.


    E. IgA and IgG paraproteins have similar serum viscosities and do not cause hyperviscosity syndrome.


QUESTION 3. A 46-year-old woman presents to your office for routine health care. She is concerned about the possibility of developing breast cancer and asks you about her risk factors. Which statement is most correct?


    A. A previous biopsy that reveals lobular carcinoma in situ (LCIS) does not substantially increase her risk of developing breast cancer.


    B. Presence of a BRCA-1 germ line mutation will substantially increase her risk of developing breast cancer.


    C. A maternal aunt with postmenopausal breast cancer will substantially increase her risk of developing breast cancer.


    D. The majority of women with breast cancer have identifiable risk factors for developing breast cancer.


    E. Duration and degree of estrogen (endogenous and exogenous) exposure are not associated with increased risk of developing breast cancer.


QUESTION 4. A 67-year-old man brought to the emergency room by his family is complaining of headaches, forgetfulness, and poor coordination. Several times over the past few weeks he has had periods of confusion and urinary incontinence. He has a history of heavy smoking and hypertension for which he takes atenolol.


    You perform an emergency computed tomography scan of the head, which reveals multiple round enhancing lesions. Chest x-ray shows a 2cm lesion in the right midlung field. The most likely diagnosis is:


    A. Prostate cancer metastatic to lung and brain


    B. Pneumonia with brain abscesses


    C. Colon cancer with lung and brain metastases


    D. Adenocarcinoma of the lung with brain metastases


    E. Gastric cancer with lung and brain metastases


QUESTION 5. A 26-year-old woman with Hodgkin lymphoma and a large mediastinal mass is treated with ABVD (doxorubicin, bleomycin, vinblastine, dacarbazine) and radiation to the mediastinum. Which of the following is true?


    A. She is more likely to die of causes other than Hodgkin lymphoma.


    B. She is not at increased risk of developing breast cancer.


    C. She has an increased risk of developing leukemia.


    D. She is not likely to remain fertile after treatment.


    E. She is not at increased risk for heart disease.


QUESTION 6. A 46-year-old woman who never smoked is diagnosed with stage IV non-small cell lung cancer, metastatic to liver and bone. Which of the following is most correct?


    A. She is potentially curable with intensive modern chemotherapy.


    B. The likelihood of responding to an epidermal growth factor receptor (EGFR) kinase inhibitor is related to the presence of a gene mutation in the intracellular portion of the kinase region.


    C. The likelihood of having a mutation in the kinase region of EGFR is random and not related to gender or ethnic background.


    D. Cytotoxic chemotherapy is the only potentially beneficial treatment.


    E. Tumors initially sensitive to kinase inhibitors do not develop resistance to these kinase inhibitors.


QUESTION 7. A 22-year-old man, previously well, is found to have a left supraclavicular mass and an otherwise normal physical examination. Chest x-ray shows bilateral paratracheal adenopathy. Fine-needle aspiration cytology of the supraclavicular mass demonstrates undifferentiated carcinoma. The next clinical action should be:


    A. Institution of multiagent chemotherapy


    B. MRI scan of the chest


    C. Mediastinoscopy and biopsy of the paratracheal nodes


    D. Testicular ultrasound


    E. Institution of radiation therapy to the mediastinum and supraclavicular areas


QUESTION 8. The following is most true about the epidemiology of lung cancer:


    A. Adenocarcinoma has become the most common histologic subtype of lung cancer.


    B. Women who smoke develop lung cancer with a similar incidence and at a similar age as do men.


    C. Asbestos does not add to the risk of developing lung cancer in smokers.


    D. Cigarette filters reduce the carcinogenic effect of cigarettes.


    E. 90% of patients with stage I non-small cell lung cancer will survive their cancer.


QUESTION 9. A 28-year-old man is admitted to the hospital with newly diagnosed acute lymphoblastic leukemia. Which of the following clinical characteristics would convey the worst prognosis?


    A. Peripheral blood blast count of 200,000/mm3


    B. T-cell phenotype


    C. Mediastinal mass


    D. Philadelphia chromosome (t9;22)


    E. Thrombocytopenia


QUESTION 10. A 51-year-old man is discovered to have a rectal cancer, which is then surgically resected. On pathology evaluation the tumor penetrates the serosa of the bowel, and one regional lymph node shows involvement with metastatic carcinoma. There is no evidence of distant metastases. Optimal therapy should include:


    A. No postoperative therapy


    B. Re-resection of pelvic tissue surrounding the area of the original tumor


    C. Radiation therapy to the pelvis


    D. Systemic chemotherapy


    E. Both radiation to the pelvis and chemotherapy

Only gold members can continue reading. Log In or Register to continue

Stay updated, free articles. Join our Telegram channel

Jul 16, 2017 | Posted by in GENERAL & FAMILY MEDICINE | Comments Off on Medical Oncology Board Review Questions

Full access? Get Clinical Tree

Get Clinical Tree app for offline access